Dole Pineapple Inc. is concerned that the 466 mL can of sliced pineapple is being overfilled. The population standard deviation is 0.88 mL. The quality control department took a random sample of 50 cans and found that the arithmetic mean volume was 467.3 mL.


At the 0.10 level of significance, can we conclude that the mean volume is greater than 466 mL?



a. What is the decision rule? (Round the final answer to 3 decimal places.)



Reject H0: μ ≤ 466 and accept H1: μ > 466 when the test statistic is
(Click to select)

.



b. What is the value of the test statistic? (Round the final answer to 2 decimal places.)



Value of the test statistic =

Answers

Answer 1

a. Decision rule: We reject the null hypothesis (μ ≤ 466) and accept the alternative hypothesis (μ > 466) if the test statistic is greater than 1.28 (from the z-table for a one-tailed test at the 0.10 level of significance).

b.The value of the test statistic is 3.75.

What is Null hypothesis?

A null hypothesis is a statement that suggests there is no significant difference between two sets of data or that any observed difference is due to chance or random sampling error.

The alternative hypothesis is a statement that contradicts the null hypothesis and proposes that there is a significant relationship between two or more variables being studied.

According to the given information:

Dole Pineapple Inc. wants to know if their 466 mL cans of sliced pineapple are being overfilled. They took a random sample of 50 cans and found the average volume to be 467.3 mL, with a known population standard deviation of 0.88 mL.

To determine if the mean volume is greater than 466 mL at a significance level of 0.10, we use a one-tailed test and the following steps:

a. Decision rule: We reject the null hypothesis (μ ≤ 466) and accept the alternative hypothesis (μ > 466) if the test statistic is greater than 1.28 (from the z-table for a one-tailed test at the 0.10 level of significance).

b. Value of the test statistic: Using the formula for the z-test, we find the test statistic to be 3.75.

Since the test statistic (3.75) is greater than the critical value (1.28), we can reject the null hypothesis and conclude that the mean volume is significantly greater than 466 mL.

To know more about Hypothesis, visit:

https://brainly.com/question/29519577

#SPJ1

Answer 2

a. Decision rule: We reject the null hypothesis (μ ≤ 466) and accept the alternative hypothesis (μ > 466) if the test statistic is greater than 1.28 (from the z-table for a one-tailed test at the 0.10 level of significance).

b.The value of the test statistic is 3.75.

What is Null hypothesis?

A null hypothesis is a statement that suggests there is no significant difference between two sets of data or that any observed difference is due to chance or random sampling error.

The alternative hypothesis is a statement that contradicts the null hypothesis and proposes that there is a significant relationship between two or more variables being studied.

According to the given information:

Dole Pineapple Inc. wants to know if their 466 mL cans of sliced pineapple are being overfilled. They took a random sample of 50 cans and found the average volume to be 467.3 mL, with a known population standard deviation of 0.88 mL.

To determine if the mean volume is greater than 466 mL at a significance level of 0.10, we use a one-tailed test and the following steps:

a. Decision rule: We reject the null hypothesis (μ ≤ 466) and accept the alternative hypothesis (μ > 466) if the test statistic is greater than 1.28 (from the z-table for a one-tailed test at the 0.10 level of significance).

b. Value of the test statistic: Using the formula for the z-test, we find the test statistic to be 3.75.

Since the test statistic (3.75) is greater than the critical value (1.28), we can reject the null hypothesis and conclude that the mean volume is significantly greater than 466 mL.

To know more about Hypothesis, visit:

https://brainly.com/question/29519577

#SPJ1


Related Questions

A trucking firm wants to purchase 10 trucks that will provide exactly 28 tons of additional shipping capacity. A model A truck holds 2 tons, a model B truck holds 3 tons, and a model C truck holds 5 tons. How many trucks of each model should the company purchase to provide the additional shipping capacity?

Answers

Answer:  Let's assume that the trucking firm purchases x trucks of model A, y trucks of model B, and z trucks of model C.

We know that the total number of trucks purchased should be 10, so:

x + y + z = 10

We also know that the total additional shipping capacity provided by the trucks should be 28 tons, so:

2x + 3y + 5z = 28

Now we have two equations with three variables. We can solve for one variable in terms of the other two, and substitute that expression into the other equation to get an equation with only two variables. For example, we can solve the first equation for x:

x = 10 - y - z

And substitute into the second equation:

2(10 - y - z) + 3y + 5z = 28

Expanding and simplifying:

20 - 2y - 2z + 3y + 5z = 28

Combining like terms:

y + 3z = 4

Now we have two equations with two variables:

x + y + z = 10

y + 3z = 4

We can solve for y in the second equation:

y = 4 - 3z

And substitute into the first equation:

x + (4 - 3z) + z = 10

Simplifying:

x + 1z = 6

x = 6 - z

Now we have three equations with three variables:

x + y + z = 10

2x + 3y + 5z = 28

x = 6 - z

We can substitute the expression for x into the first equation:

(6 - z) + y + z = 10

Simplifying:

y = 4 - (6 - z)

y = z - 2

Now we have expressed all three variables in terms of z. We can substitute these expressions into the second equation and solve for z:

2(6 - z) + 3(z - 2) + 5z = 28

Simplifying:

12 - 2z + 3z - 6 + 5z = 28

Combining like terms:

6z + 6 = 28

Solving for z:

z = 3

Now we can use the expressions for x and y to find how many trucks of each model the company should purchase:

x = 6 - z = 3

y = z - 2 = 1

Therefore, the company should purchase 3 trucks of model A, 1 truck of model B, and 6 trucks of model C to provide exactly 28 tons of additional shipping capacity.

Step-by-step explanation:

What is the perimeter of the trapezoid?

Answers

40
use pythagoras theorem to find the diagonal side: 8^2+6^2=100 so it’s 10 then add the other sided together

tanya has a dog leash that is 4 yards long. she shortens the leash by 6 feet.what is the lenght of shortened leash

Answers

Answer:

2 yards or 6 feet long

Step-by-step explanation:

Each yard is 3 feet long.

If Tanya shortens the leash by 6 feet, that is 2 yards.

4-2=2

The leash is 2 yards (or 6 feet) long.

Answer:

6 foot

Step-by-step explanation:

1 yards = 3 foot

4 yards = 12 foot

We take

12 - 6 = 6 foot

So, the length of the shortened least is 6 foot.

"The quotient of 30 and a number is decreased by 2." please help

Answers

This sentence relating to the quotient can be expressed mathematically as:

(30 / x) - 2

What is the explanation for the above response?


This sentence can be expressed mathematically as:

(30 / x) - 2

where x represents the unknown number.

The word "quotient" indicates that we are dividing 30 by the unknown number x. The phrase "is decreased by 2" means that we need to subtract 2 from the quotient.

Learn more about quotient  at:

https://brainly.com/question/16134410

#SPJ1

Which choice is NOT equal to the others? Responses A −[[2/5]]−[[2/5]] B [[2/−5]][[2/−5]] C [[−2/5]][[−2/5]] D [[2/5]]

Answers

Answer:

B is the answer

Step-by-step explanation:

The expression that is not equal to the others is B [[2/−5]] The other expressions are A −[[2/5]], C [[−2/5]], and D [[2/5]].

Use circle K for problems 11-13

Answers

After calculation we get

11) d. m/JML = 90 degrees, a. m/GMH = 45 degrees, b. mLH = 90 degrees, c. m/LKH = 45 degrees

12) a. AGKH is an isosceles triangle, b. Another triangle formed by two radii is an equilateral triangle.

13) a. m/GKH = 33.4 degrees, c. m/KHJ = 33.4 degrees, e. mJL66.8 degrees, b. m/KGH = 66.8 degrees, d. m/JKL = 113.2 degrees

What is quadrilateral?

A quadrilateral is a geometric shape with four straight sides and four angles. Examples of quadrilaterals include squares, rectangles, parallelograms, trapezoids, and kites.

According to the given information:

a. Since m/LGH and m/GHJ are both 45°, GH is a bisector of ∠JGL. Therefore, m/GMH = 90° - 45° = 45°.

b. Since GH is a bisector of ∠JGL, m/LGH = m/HGL = 45°. Also, since LH is a straight line, m/LGH + mLH + m/HGL = 180°. Thus, mLH = 90° - 45° = 45°.

c. Since GH is a bisector of ∠LJK, m/GHK = m/JHK = 45°. Also, since LK is a straight line, m/LKH + m/JHK + m/LJK = 180°. Thus, m/LKH = m/LJK = (180° - 2*45°)/2 = 45°.

d. Since GH is a bisector of ∠JGL and JML is a straight line, m/JML = m/JGH + m/HGL = 45° + 45° = 90°.

a. AGKH is a quadrilateral with two sides that are radii of the circle. Since all radii of a circle are equal, AGKH is a kite. Furthermore, since the two radii AG and KH are perpendicular to each other, AGKH is also a rectangle.

b. Another triangle formed by two radii is AKJ, where AK and AJ are radii of the circle and KJ is a chord.

a. Since GH is a diameter of the circle and GKH is a right triangle with ∠GHK = 90°, m/GKH = 180° - 90° = 90°.

b. Since GH is a diameter of the circle and KJ is a chord, m/KGH = m/KJ = 1/2 * m/KHJ = 1/2 * (180° - 113.2°) = 33.4°.

c. Since GH is a diameter of the circle and KHJ is a right triangle with ∠KHJ = 90°, m/KHJ = 180° - m/GKH = 180° - 90° = 90°.

d. Since GH is a diameter of the circle and JKL is a right triangle with ∠JKL = 90°, m/JKL = 180° - m/KJ - m/JKH = 180° - 33.4° - 45° = 101.6°.

e. Since JL is a chord of the circle and ∠JGL is an inscribed angle that intercepts it, m/JGL = 1/2 * m/JL = 1/2 * (180° - m/JKL) = 1/2 * (180° - 101.6°) = 39.2°. Also, since GH is a diameter of the circle and GJL is a right triangle with ∠GJL = 90°, m/GJL = 90° - m/GHL = 90° - 45° = 45°. Therefore, m/JLH = m/JGL - m/GLH = 39.2° - 45° = -5.8°. Note that the negative value indicates that ∠JLH is a reflex angle.

To know more about quadrilateral visit:

https://brainly.com/question/12431044

#SPJ1

PLEASE HELPPPPPP ME PLEASE

Answers

If the the a is greater than 1, compared to the parent function the C. Stretched vertically.

How to find the comparison ?

The equation y = ax^2 + c represents a quadratic function where "a" is the coefficient of the x^2 term and "c" is a constant term. The parent function of this quadratic function is y = x^2.

If the equation of a quadratic function is given in the form y = ax^2 + c and "a" is greater than 1, then the graph of the function will be stretched vertically and the vertex will be shifted up or down depending on the value of "c".

Find out more on parent functions at https://brainly.com/question/17079244

#SPJ1

A large production facility uses two machines to produce a key part for its main product. Inspectors have expressed concern about the quality of the finished product. Quality-control investigation has revealed that the key part made by the two machines is defective at times. The inspectors randomly sampled 35 units of the key part from each machine. Of those produced by machine A, 5 were defective. Seven of the 35 sampled parts from machine B were defective. The production manager is interested in estimating the difference in proportions of the populations of parts that are defective between machine A and machine B. From the sample information, compute a 98% confidence interval for this difference.

Answers

The range of the 98% confidence interval for the percentage of faulty components that differ between machines A and B is about between -0.2448 and 0.1305. (rounded to 4 decimal places).

How can you figure up a confidence interval for the proportional difference?Define the populations of interest and the characteristic you want to compare (e.g., proportion of success or failure).Collect random samples from each population and record the number of occurrences of the characteristic of interest in each sample.Calculate the sample proportions ([tex]p_1 \;and \;p_2[/tex]) by dividing the number of occurrences by the sample size for each population.Calculate the standard error (SE) of the difference in proportions using the sample proportions, sample sizes, and appropriate formula (SE = [tex]\sqrt{ [(p1 * (1 - p1) / n1) + (p2 * (1 - p2) / n2)}[/tex], where[tex]p_1 \;and \;p_2[/tex] are the sample proportions and[tex]n_1 \;and \;n_2[/tex] are the sample sizes for the two populations, respectively).Determine the appropriate critical value from the probability distribution (e.g., standard normal distribution for large sample sizes or t-distribution for small sample sizes) based on the desired confidence level.Calculate the margin of error (ME) by multiplying the standard error by the critical value (ME = critical value * SE).Construct the confidence interval by adding and subtracting the margin of error from the sample statistic (e.g., the difference in sample proportions, or the ratio of sample proportions).Interpret the confidence interval, stating that we can be [confidence level]% confident that the true population parameter falls within the calculated interval.

Given:

Sample proportion from machine A ([tex]p_1[/tex]) = 5/35 = 0.14285714285714285

Sample proportion from machine B ([tex]p_2[/tex]) = 7/35 = 0.2

Sample size from machine A ([tex]n_1[/tex]) = 35

Sample size from machine B ([tex]n_2[/tex]) = 35

Standard error (SE) = [tex]\sqrt{ [(p1 * (1 - p1) / n1) + (p2 * (1 - p2) / n2)}[/tex]

= [tex]\sqrt{[0.14285714285714285 * (1 - 0.14285714285714285) / 35] + [0.2 * (1 - 0.2) / 35] }[/tex]

= 0.07058061453775912 (rounded to 11 decimal places)

Margin of error (ME) = Critical value * Standard error

= 2.660 * 0.07058061453775912 (using z-score for a 98% confidence level)

= 0.18765117789861733 (rounded to 11 decimal places)

Confidence interval (CI) = Sample statistic ± Margin of error

= [tex](p_1 - p_2) \pm ME[/tex]

= (0.14285714285714285 - 0.2) ± 0.18765117789861733

= -0.05714285714285715 ± 0.18765117789861733

The 98% confidence interval for the difference in proportions of defective parts between machine A and machine B is approximately -0.2448 to 0.1305 (rounded to 4 decimal places).

Learn more about confidence interval here:

https://brainly.com/question/29680703

#SPJ1

Can someone help me with this problem? I need to find x and y

Answers

Answer:

x = √17

y = 10.1

Step-by-step explanation:

x² + 8² =  9²

x² = 81 - 64 = 17

x = √17

sin∅ = √17/9

∅ = 27.27°

9/y = cos(27.27)

y = 9/cos(27.27) = 10.13

y = 10.1

How to solve it and the answer

Answers

Answer: The answer is, y=18x

Step-by-step explanation: This is going up by 18 as the y axis increases by 9. Hope this helped :)

Trangle ABC has an area 25 square feet and perimeter of 65.5 feet of triangle ABC is dilated by a factor of 5/2 to create now calculate the area of trangle DEF using the scale factor

Answers

So, the area of triangle DEF is 312.5 square feet, using the scale factor of 5/2.

What is dilation?

the context of mathematics and geometry, dilation is a transformation that changes the size of an object. It is a type of transformation that scales an object by a certain factor, without changing its shape or orientation.

In other words, dilation involves multiplying the coordinates of a geometric figure by a fixed constant, which results in an enlarged or reduced version of the original figure. The constant is known as the dilation factor or the scale factor, and it can be any real number greater than zero.

For example, if we dilate a circle by a scale factor of 2, every point on the circle will be moved twice as far away from the center, resulting in a new circle with a diameter twice as large as the original.

Let's start by using the formula for the perimeter of a triangle:

[tex]Perimeter of triangle ABC = AB + BC + AC = 65.5 feet[/tex]

We can also use Heron's formula to find the area of triangle ABC:

[tex]Area of triangle ABC = \sqrt(s(s-AB)(s-BC)(s-AC))[/tex]

where s is the semi perimeter of the triangle:

[tex]s = (AB + BC + AC) / 2[/tex]

We can use these equations to solve for the side lengths of triangle ABC:

[tex]AB + BC + AC = 65.5[/tex]

[tex]s = (AB + BC + AC) / 2[/tex]

[tex]25 = \sqrt(s(s-AB)(s-BC)(s-AC))[/tex]

Solving for AB, BC, and AC gives us:

AB = 15

BC = 20

AC = 30.5

Now, let's dilate triangle ABC by a factor of 5/2 to create triangle DEF. This means that each side of triangle ABC will be multiplied by 5/2 to get the corresponding side length of triangle DEF.

DE = AB * (5/2) = 37.5

EF = BC * (5/2) = 50

DF = AC * (5/2) = 76.25

Now we can use Heron's formula again to find the area of triangle DEF:

s = (DE + EF + DF) / 2 = 81.875

Area of triangle DEF = sqrt(s(s-DE) (s-EF) (s-DF)) = 312.5 square feet

To know more about triangle, visit:

https://brainly.com/question/2773823

#SPJ1

Becky can make a monthly payment of $530 for a car. If
the annual interest rate she qualifies for is 6% for 4 years,
what price could she afford for the car?

Answers

Becky can afford a car with a price of $20,858.33 or less, given her monthly payment of $530, and assuming an annual interest rate of 6% for 4 years.

What is statistics?

Statistics is a branch of mathematics that deals with the collection, analysis, interpretation, presentation, and organization of numerical data.

To calculate the price that Becky can afford for the car, we need to use the present value formula for an annuity:

PV = PMT x [tex]((1 - (1 + r/n)^{(-nt))}[/tex] / (r/n))

where:

PV = present value of the car

PMT = monthly payment

r = annual interest rate

n = number of compounding periods per year

t = number of years

In this case, PMT = $530, r = 6% (or 0.06 as a decimal), n = 12 (since monthly payments are being made), and t = 4. Plugging in these values, we get:

PV = $530 x[tex]((1 - (1 + 0.06/12)^{(-12*4)})[/tex] / (0.06/12))

PV = $20,858.33

Therefore, Becky can afford a car with a price of $20,858.33 or less, given her monthly payment of $530, and assuming an annual interest rate of 6% for 4 years.

To learn more about statistics from the given link:

https://brainly.com/question/28053564

#SPJ1

please help <3 gracias

Answers

The function evaluated in x = 2 is:

3f(2) =12

How to evaluate the function?

Here we know that:

f(x) = x^2

And we want to evaluate the expression:

3f(2)

To do that replace x by 2.

3f(2) = 3*(2^2) = 3*4 = 12

That is the value of the expression.

Learn more about evaluating functions at:

https://brainly.com/question/1719822

#SPJ1

Translate in two ways each of these statements into logical expressions using predcates quantifiers and logical connective first let the domain consist of the students in your class and second let it consist of all people a) everyone in your class has a cellular phone

Answers

For all x, P(x) (using universal quantifier ∀) and  It is not the case that there exists an x such that ~P(x) (using negation ¬ and existential quantifier ∃)

What is probability?

Probability is a measure of the likelihood of an event occurring. It is a number between 0 and 1, where 0 means the event is impossible and 1 means the event is certain to happen.

Let S be the set of students in your class and P(x) be the predicate "x has a cellular phone". Then, we can represent the statement "everyone in your class has a cellular phone" as:

1. For all x in S, P(x) (using universal quantifier ∀)

2. It is not the case that there exists an x in S such that ~P(x) (using negation ¬ and existential quantifier ∃)

If we want to represent the same statement for all people, we can use the same predicate P(x) and consider the domain of all people. Then, the statement "everyone has a cellular phone" can be represented as:

For all x, P(x) (using universal quantifier ∀)

It is not the case that there exists an x such that ~P(x) (using negation ¬ and existential quantifier ∃)Let S be the set of students in your class and P(x) be the predicate "x has a cellular phone". Then, we can represent the statement "everyone in your class has a cellular phone" as:

For all x in S, P(x) (using universal quantifier ∀)

It is not the case that there exists an x in S such that ~P(x) (using negation ¬ and existential quantifier ∃)

If we want to represent the same statement for all people, we can use the same predicate P(x) and consider the domain of all people. Then, the statement "everyone has a cellular phone" can be represented as:

1. For all x, P(x) (using universal quantifier ∀)

2. It is not the case that there exists an x such that ~P(x) (using negation ¬ and existential quantifier ∃)

To learn more about probability from the given link:

https://brainly.com/question/30034780

#SPJ1

What number is halfway between 10 and 17

Answers

Answer:

13.5

Step-by-step explanation:

10+17=27

27/2=13.5

Answer:

13.5

Step-by-step explanation:

What number is halfway between 10 and 17

State any domain restrictions for the expression below from least to greatest (for example: -2,-1,0,1,2), by using one answer box for each domain restriction, then simplify the expression in the last answer box. (81-x²) (x² + 2x − 63) 2x² - 6x 3x2 30x + 63 3x 81x² ÷​

Answers

The domain restrictions on the function [tex]\left f(x\right)=-\frac{3x}{81\:-\:x^2}\:\cdot \frac{81\:-\:x^2}{2x^2-6x}\:\div \frac{x^2+2x-6x}{3x^2-30x+63}[/tex] are the x values -9, 0, 3, 4 and 9


Calculating the domain restriction

From the question, we have the following function that can be used in our computation:

[tex]\left f(x\right)=-\frac{3x}{81\:-\:x^2}\:\cdot \frac{81\:-\:x^2}{2x^2-6x}\:\div \frac{x^2+2x-6x}{3x^2-30x+63}[/tex]

Next, we set the denominator to 0 and solve for x

So, we have

81 - x²: x = ±9

2x² - 6x: x = 0 and x = 3

x² + 2x - 6x: x = 0 and x = 4

Hence, the domain restrictions are the x values -9, 0, 3, 4 and 9

Read more about domain at

https://brainly.com/question/2264373

#SPJ1

Tyrone has $800 in a savings account that earns 10% annually. The interest is not compounded. How much will he have in total in 1 year?

Answers

Answer: $80

Step-by-step explanation:

If Tyrone's savings account earns a simple 10% annual interest and is not compounded, then the interest for one year can be calculated using the formula:

Interest = Principal × Rate × Time

where:

Principal = $800

Rate = 10% = 0.1

Time = 1 year

Interest = $800 × 0.1 × 1 = $80

Tyrone will earn $80 in interest in 1 year. To find the total amount in his account after 1 year, we add the interest to the principal:

Total amount = Principal + Interest

Total amount = $800 + $80 = $880

In 1 year, Tyrone will have $880 in total in his savings account.

Answer:

$880

Step-by-step explanation:

10% = .1

800 x .1 = 80

80 + 800= 880

question attached
A. 5
B. 16
C. function is not defined for this value
D. 9

Answers

F(4)=9 is the value for this function.

What is piecewise function?

A function that is defined by numerous smaller functions across various time intervals is known as a piecewise function. The domain of a function is the sum of all the smaller domains, and each sub-function has its own formula and domain. The input value and the function that establishes that interval determine the function's output.

The typical functional notation, which represents the body of a function as an array of functions and related subdomains, can be used to define piecewise functions. Together, these subdomains must encompass the entire domain; frequently, it is also necessary for them to be pairwise disjoint, or constitute a partition of the domain.

x=4 is bigger than or equal to 0 because it.

we use the third function definition

F(x)=x+5.

Therefore, F(4)=4+5=9.

To know more about function visit:

brainly.com/question/12431044

#SPJ1

The numbers of students in the 9 schools in a district are given below.
(Note that these are already ordered from least to greatest.)
164, 225, 227, 250, 261, 268, 277, 379, 523
Send data to calculator
Suppose that the number 523 from this list changes to 424. Answer the following.
(a) What happens to the mean?
(b) What happens to the median?
It decreases by
O It increases by 0.
It stays the same.
O It decreases by 0.
It increases by
It stays the same.,
X
5

Answers

if we change the value of 523 to 424 in the list of numbers, then the mean decreases by approximately 3.22 and the median stays the same.

How to calculate the mean?

To calculate the mean, we add up all the numbers in the list and divide by the total number of values. Before the change, the sum of the numbers is:

164 + 225 + 227 + 250 + 261 + 268 + 277 + 379 + 523 = 2494

And there are 9 numbers in the list. So the mean is:

2494 / 9 ≈ 277.11

If we change the value of 523 to 424, then the sum becomes:

164 + 225 + 227 + 250 + 261 + 268 + 277 + 379 + 424 = 2465

And there are still 9 numbers in the list. So the new mean is:

2465 / 9 ≈ 273.89

So the mean decreases by approximately 3.22.

To calculate the median, we find the middle value of the list. If the list has an odd number of values, then the median is the middle value. If the list has an even number of values, then the median is the average of the two middle values. In this case, the list has an odd number of values, so the median is:

261

If we change the value of 523 to 424, then the list becomes:

164, 225, 227, 250, 261, 268, 277, 379, 424

And the median is still:

261

So the median stays the same.

In summary, if we change the value of 523 to 424 in the list of numbers, then the mean decreases by approximately 3.22 and the median stays the same.

To know more about mean visit :-

https://brainly.com/question/26177250

#SPJ1

The table shows the monthly rainfall at a measuring station.

Month Rainfall
(inches) Month Rainfall
(inches)
Jan 2.22 Jul 3.37
Feb 1.51 Aug 5.40
Mar 1.86 Sep 5.45
Apr 2.06 Oct 4.34
May 3.48 Nov 2.64
Jun 4.47 Dec 2.14
a. What is the mean monthly rainfall? Round your answer to the nearest thousandth.

Answers

The mean monthly rainfall from the given data is 3.245 inches, using the formula to evaluate mean that is sum of all observations divided by total number of observations.

What is mean?

By dividing the sum of the given numbers by the entire number of numbers, the mean—the average of the given numbers—is calculated. Similar to the mode and median, the mean is one of the measurements of central tendency. It denotes that values for a specific set of data are distributed equally. The three most frequently employed measures of central tendency are the mean, median, and mode. The total values provided in a datasheet must be added, and the sum must be divided by the total number of values in order to determine the mean.When all of the values are organised in ascending order, the Median is the median value of the given data. While the number in the list that is repeated a maximum of times is the mode.

Mean= [tex]\frac{sum of all observations }{total observations}[/tex]

Given observations in inches:

2.22, 1.51 , 1.86 , 2.06 , 3.84 , 4.47 , 3.37,5.40 ,5.45 , 4.34 ,2.64,2.14

sum of all observations = 2.22 + 1.51 + 1.86 + 2.06 + 3.84 + 4.47 + 3.37 + 5.40 + 5.45 + 4.34 + 2.64 + 2.14

sum of all observations = 38.94 inches

Total number of observations = total number of months

                                                = 12

Mean monthly rainfall= [tex]\frac{sum of all observations }{total observations}[/tex]

                                   =[tex]\frac{38.94}{12}[/tex]

                                   =3.245

Mean monthly rainfall=3.245 inches.

To know more about mean , visit:

https://brainly.com/question/31101410

#SPJ1

Refer to the attachment for the table.

y varies directly as the cube of x. when x= 4, then y= 7. find y when x=5

Answers

When X = 5, Y is approximately equal to 27.34.

What is proportion?

The size, number, or amount of one thing or group as compared to the size, number, or amount of another. The proportion of boys to girls in our class is three to one.

We are given that "Y varies directly as the cube of X", which can be written as:

Y = kX³

where k is a constant of proportionality. We need to find the value of k to solve for Y when X = 5.

Using the values given in the problem, we can write:

7 = k(4³)

Simplifying this equation, we get:

7 = 64k

Dividing both sides by 64, we get:

k = 7/64

Now that we know the value of k, we can solve for Y when X = 5:

Y = (7/64)(5³) = 27.34 (rounded to two decimal places)

Therefore, when X = 5, Y is approximately equal to 27.34.

To learn more about the proportion visit:

brainly.com/question/870035

#SPJ1

Tom is preparing for a 100 meters race competition. During his practice last week, a sample of seven 100
meters races is reviewed, and the finishing times (in seconds) were as below:
13.4
15.6
13.1
14.5
14.2
13.3
15.3
It is reasonable to assume his finishing times are normally distributed.
(a) Construct a 99% confidence interval estimate of his population mean finishing time of 100 meters race.
(b) If the confidence interval estimate of his population mean finishing time of 100 meters is constructed at
95% instead of 99%, would the new confidence interval be (I) wider, (II) narrower, or (III) the same as
the interval constructed at part (a)? (Just state your answer, no calculation is needed in part (b))

Answers

Question A) A 99% confidence interval estimate of his population mean finishing time of 100 meters race is (12.8081,15.5919)

Question B) Option II) narrower is the correct Option.

What is  linear equation?

An algebraic equation with simply a constant and a first- order( direct) element, similar as y = mx b, where m is the pitch and b is the y- intercept, is known as a linear equation.

                         The below is sometimes appertained to as a" direct equation of two variables," where y and x are the variables. Equations whose variables have a power of one are called direct equations. One illustration with only one variable is where layoff b = 0, where a and b are real values and x is the variable.

Let X be the time required to finish 100 meters race competition with Tom ( in seconds.)

A) A 99% confidence interval estimate of his population mean finishing time of 100 meters race is (12.8081,15.5919)

x = 14.2

s^2= 0.9867

s= 0.9933

α/2, n-1 = 3.7074

Margin of error= 1.3919

lower bound= 12.8081

upper bound= 15.5919

Learn more about Linear equation

brainly.com/question/11897796

#SPJ1

Dravin is painting a cone -shaped centerpiece for the school dance . The centerpiece had a diameter of 12 inches and slant height of 19 inches . What is the total surface area that needs painting? Round your answer to the nearest whole number

Answers

The total surface area that needs painting is approximately 487 square inches.

What is the cone total surface area?

The total surface area (TSA) of a cone is given by the formula:

TSA = πr(r + l)

The surface area of a cone can be calculated using the formula:

[tex]Surface Area = \pi r(r + \sqrt{(r^2 + h^2)} )[/tex]

where r is the radius of the base of the cone and h is the slant height of the cone.

Given:

Diameter of the cone = 12 inches

Radius (r) = Diameter / 2 = 12 / 2 = 6 inches

Slant height (h) = 19 inches

Plugging in the values into the formula, we get:

[tex]Surface Area = \pi * 6 * (6 + \sqrt{(6^2 + 19^2)} )[/tex]

Calculating the value inside the square root:

[tex]\sqrt{(6^2 + 19^2)} = \sqrt{(36 + 361) } = \sqrt{397} = 19.92[/tex] (rounded to two decimal places)

Substituting the value back into the formula:

[tex]Surface Area = \pi * 6 * (6 + 19.92) = \pi * 6 * 25.92 = 487.34[/tex] square inches (rounded to two decimal places)

Hence, the total surface area that needs painting is approximately 487 square inches.

To learn more about cone total surface area, Visit

https://brainly.com/question/27812847

#SPJ1

Owen has two options for buying a car. Option A is 1.3 % APR financing over 36 months and Option B is 5.2 % APR over 36 months with $1500 cash back, which he
would use as part of the down payment. The price of the car is $32,020 and Owen has saved $3200 for the down payment. Find the total amount Owen will spend on the
car for each option if he plans to make monthly payments. Round your answers to the nearest cent, if necessary.
Option A:
Option B:

Answers

Answer: Option A:

To calculate the total amount Owen will spend on Option A, we need to calculate the monthly payment and then multiply it by the number of months:

First, we need to calculate the total amount of the loan. Owen is making a down payment of $3200, so he will be borrowing $28,820 (the price of the car minus the down payment).

Next, we can use the formula for calculating the monthly payment for a loan:

P = (r * A) / (1 - (1 + r)^(-n))

where P is the monthly payment, r is the monthly interest rate, A is the total amount of the loan, and n is the number of months.

For Option A, the monthly interest rate is 1.3% / 12 = 0.01083, the total amount of the loan is $28,820, and the number of months is 36. Plugging these values into the formula, we get:

P = (0.01083 * 28,820) / (1 - (1 + 0.01083)^(-36)) = $860.45

Therefore, the total amount Owen will spend on Option A is:

36 * $860.45 = $30,975.98

Option B:

For Option B, we need to take into account the $1500 cash back that Owen will receive as part of the down payment. This means that the total amount of the loan will be $32,020 - $3200 - $1500 = $27,320.

To calculate the monthly payment, we can use the same formula as before:

P = (r * A) / (1 - (1 + r)^(-n))

For Option B, the monthly interest rate is 5.2% / 12 = 0.04333, the total amount of the loan is $27,320, and the number of months is 36. Plugging these values into the formula, we get:

P = (0.04333 * 27,320) / (1 - (1 + 0.04333)^(-36)) = $825.53

Therefore, the total amount Owen will spend on Option B is:

36 * $825.53 + $1500 = $30,316.08

Therefore, Option A will cost Owen a total of $30,975.98, and Option B will cost him a total of $30,316.08. Therefore, Option B is the cheaper option for Owen.

Step-by-step explanation:

Write the equation in standard form for the circle with center (0,5) and radius 7.

Answers

Answer:

[tex]x^2+(y-5)^2=49[/tex]

Step-by-step explanation:

Recall the formula for the graph of a circle:

[tex](x-h)^2+(y-k)^2=r^2\\[/tex]

Where h is the x-coordinate of the vertex, k is the y-coordinate of the vertex, and r is the radius.

We are given the vertex and the length of the radius.

Substitute the values:

[tex](x-0)^2+(y-5)^2=7^2=\\x^2+(y-5)^2=49[/tex]

Thus, the standard form is:

[tex]x^2+(y-5)^2=49[/tex]

please help me in this question​

Answers

Answer:

step by step explanation:

All you have to do is expand and reduce the expressions

then evaluate -2 being a root of the expressions

To do that you need to substitute -2 into the simplified expressions

if the result comes as zero then f(-2) is factor of f(x) according to the factor theorem.

Example 1.

simplify (-5x-2)(7x-4)-2x+3

if you substitute f(x) as f(-2)

then substitute x with -2

when you simply and evaluate the expression you will get that the expression is equal to -137

which means -2 isn't a root since the expression must be equal to 0

-2 is not a root

do the same for the other expressions

Each of forty-nine moviegoers was asked, "What is your favorite movie type?"
Here are the results.
12 men and 10 women chose "Drama".
. 14 men and 13 women chose "Action".
Construct a two-way frequency table for the data.

Answers

The numbers in each cell represent the frequency of people who chose that combination of gender and movie type.

What is the frequency?

The number of periods or cycles per second is called frequency. The SI unit for frequency is the hertz (Hz). One hertz is the same as one cycle per second.

To construct a two-way frequency table for the data, we need to organize the responses by gender and movie type. The table is in the attached image.

The rows represent the gender of the moviegoers, and the columns represent the movie type.

The numbers in each cell represent the frequency of moviegoers who chose that combination of gender and movie type.

For example, there are 12 men who chose Drama as their favorite movie type, and there are 13 women who chose Action as their favorite movie type.

The total number of moviegoers who chose Drama is 22, and the total number who chose Action is 27. The total number of moviegoers is 49.

Hence, The numbers in each cell represent the frequency of people who chose that combination of gender and movie type. For example, 12 men chose Drama as their favorite movie type.

To know more about the frequency visit:

https://brainly.com/question/5102661

#SPJ1

Kyd and North are playing a game. Kyd selects one card from a standard 52-card deck. If Kyd selects a face card (Jack, Queen, or King), North pays him $6. If Kyd selects any other type of card, he pays North $3.

Answers

Kyd's expected value is positive and North's expected value is negative, Kyd has the advantage in this game

What is expected value?

Expected value is a concept used in probability theory to describe the average value or outcome of a random variable over many trials. It is calculated by multiplying each possible outcome of a variable by its probability, and then summing all of the products. This can help to estimate the most likely outcome or value of an uncertain event or situation.

The probability of selecting a face card from a standard 52-card deck is 12/52 or 3/13. The probability of selecting any other type of card is 1 - 3/13 or 10/13.

Kyd's expected value for this game can be calculated as follows:

(3/13) x (-$3) + (10/13) x $6 = $3.38

Therefore, Kyd's expected value for this game is $3.38.

North's expected value for this game can also be calculated as follows:

(3/13) x $6 + (10/13) x (-$3) = -$0.92

Therefore, North's expected value for this game is -$0.92.

Since Kyd's expected value is positive and North's expected value is negative, Kyd has the advantage in this game.

To learn more about expected value visit the link:

https://brainly.com/question/24305645

#SPJ1

What is the circumference of the circle? Use 3.14 for π
. Round your answer to the nearest hundredth. please help 100 points

Answers

Answer:

b)

Step-by-step explanation:

50 Points! Multiple choice algebra question. Shen is simplifying the expression (3x^4+4x^2) (x^3-2x^2-1). Which of the following shows the correct product. Photo attached. Thank you!

Answers

So, multiple choice algebra questions. the correct answer would be option D: [tex]3x^7 - 6x^6 - 11x^4 + 4x^5 - 4x^2[/tex].

To simplify the given expression [tex](3x^4+4x^2) (x^3-2x^2-1)[/tex], we can use the distributive property of multiplication to multiply each term of the first expression by each term of the second expression. This gives us:

[tex](3x^4+4x^2) (x^3-2x^2-1) \\= 3x^4(x^3) + 3x^4(-2x^2) + 3x^4(-1) + 4x^2(x^3) - 4x^2(2x^2) - 4x^2(1)[/tex]

Simplifying each term, we get:

[tex]= 3x^7 - 6x^6 - 3x^4 + 4x^5 - 8x^4 - 4x^2[/tex]

So, the correct answer would be option D: [tex]3x^7 - 6x^6 - 11x^4 + 4x^5 - 4x^2.[/tex]

To know more about distributive property visit:

https://brainly.com/question/6276874

#SPJ1

Other Questions
true or false: a lease is an annuity when it requires equal payments at the same interval. true false question. true false a ? is a wheel with a concave edge for supporting a moving rope that is changing direction. suppose riley trained two neural networks n1 and n2 on the same training data. to decide which network to use at test time, riley proposes to pick the network with the smallest training loss. amari argues that this is a bad idea. who is right? explain briefly. a vertical laminar flow hood should be cleaned: select one: a. starting on the right, wiping side to side and working back away from the user. b. starting on the left, wiping side to side and working back away from the user. c. starting in the front, wiping side to side and working back away from the user. d. starting in the back, wiping side to side and working forward toward the user. Where did the happy elf put the golden coins? what drug can be used to control ventricular rate in a patient with atrial fibrillation naplex A football kicker attempted to make field goals from different distances from the goal post. The relationship between the distance from the goal post and the number of field goals made is shown in the scatter plot.Which of the following tables of data represents the scatter plot based on the questions on the image below? !!ill give brainlist!!Tim is taking the path shown to bike to Greg's house. Tim travels an average speed of 15 mi/hr.Determine how much time it will take Tim to get to Greg's house. (You may use decimals here) John is making his pasta dish. his mom eats 1/12 of pasta and his brother eats 1/6 of pasta. How much of pasta is left over? would it be ethical to charge campers different rates depending on their family's socioeconomic status identify a characteristic of change managers that an organization should consider when employing people.Change managers are nonconformists who take risks.Having a large number of conformist is effective for an organization.Change managers are usually regarded as peacemakers who follow rules.Having a large number of radical innovators is effective for an organization. A client is complaining of pain in the right upper quadrant and also in the right shoulder. Which organ would the nurse suspect as being involved?a)Stomachb)Gall bladderc)Pancreasd)Kidneys Corporation X can issue straight 5-year debt (bonds) at a yield to maturity of 5%. If a 5-year at-the-money call option on the S&P 500 index costs 20% of the index value, what percentage of the indexs upside over the next 5 years could a 5-year structured note issued by Corporation X provide, assuming a 2% up-front underwriting spread? The Clean Water Act is inapplicable to ponds that are not adjacent to open water.True/false Douglas Keel, a financial analyst for Orange Industries, wishes to estimate the rate of return for two similar-risk investments, X and Y. Douglas's research indicates that the immediate past returns will serve as reasonable estimates of future returns. A year earlier, investment X had a market value of $20,000; investment Y had a market value of $55,000. During the year, investment X generated cash flow of $1,500 and investment Y generated cash flow of $6,800. The current market values of investments X and Y are $21,000 and $55,000, respectively. a. Calculate the expected rate of return on investments X and Y using the most recent years data. b. Assuming that the two investments are equally risky, which one should Douglas recommend? Why? A rectangular prism is shown in the image.A rectangular prism with dimensions of 5 yards by 5 yards by 3 and one half yard.What is the volume of the prism? twenty eight and one half yd3 forty one and one fourth yd3 eighty seven and one half yd3 166 yd3 a products table and an orders table have several linked records that are joined by a cross-referencing table named productsorders. what kind of table relationship do these tables demonstrate? Why should a university give you $10,000? and how would you spend it as a wise college student? What does Gregor's room tell about Gregor? A city charter most closely resemblesa. a budget.b. a constitution.c. a levy.d. an ordinance.